Does changing the order of the indices of the Kronecker delta within a summation matter?

The name of the pictureThe name of the pictureThe name of the pictureClash Royale CLAN TAG#URR8PPP











up vote
1
down vote

favorite
2












In my notes I have that




$$sum_m a_m delta_nm=a_1delta_n1+a_2delta_n2+a_3delta_n3+cdots=a_ntagA$$




Is this really correct?



I thought that for the Kronecker delta the first index must match the summation index. For this reason, I thought that the expression $(mathrmA)$ should written as



$$sum_colorbluem a_m delta_colorbluemn=a_1delta_1n+a_2delta_2n+a_3delta_3n+cdots=a_ntagB$$



Just to make my argument clear, I have made the color of the indices match.



After looking at this page on the Kronecker delta I know that it is okay to write
$$sum_ja_jdelta_ij=a_itag1$$
or
$$sum_ia_idelta_ij=a_jtag2$$



Expression $(1)$ matches $(mathrmA)$ and expression $(2)$ matches $(mathrmB)$




So does this mean that switching the order of the indices of the Kronecker delta (within a summation) has no effect on the result (the RHS), or am I missing something?










share|cite|improve this question

























    up vote
    1
    down vote

    favorite
    2












    In my notes I have that




    $$sum_m a_m delta_nm=a_1delta_n1+a_2delta_n2+a_3delta_n3+cdots=a_ntagA$$




    Is this really correct?



    I thought that for the Kronecker delta the first index must match the summation index. For this reason, I thought that the expression $(mathrmA)$ should written as



    $$sum_colorbluem a_m delta_colorbluemn=a_1delta_1n+a_2delta_2n+a_3delta_3n+cdots=a_ntagB$$



    Just to make my argument clear, I have made the color of the indices match.



    After looking at this page on the Kronecker delta I know that it is okay to write
    $$sum_ja_jdelta_ij=a_itag1$$
    or
    $$sum_ia_idelta_ij=a_jtag2$$



    Expression $(1)$ matches $(mathrmA)$ and expression $(2)$ matches $(mathrmB)$




    So does this mean that switching the order of the indices of the Kronecker delta (within a summation) has no effect on the result (the RHS), or am I missing something?










    share|cite|improve this question























      up vote
      1
      down vote

      favorite
      2









      up vote
      1
      down vote

      favorite
      2






      2





      In my notes I have that




      $$sum_m a_m delta_nm=a_1delta_n1+a_2delta_n2+a_3delta_n3+cdots=a_ntagA$$




      Is this really correct?



      I thought that for the Kronecker delta the first index must match the summation index. For this reason, I thought that the expression $(mathrmA)$ should written as



      $$sum_colorbluem a_m delta_colorbluemn=a_1delta_1n+a_2delta_2n+a_3delta_3n+cdots=a_ntagB$$



      Just to make my argument clear, I have made the color of the indices match.



      After looking at this page on the Kronecker delta I know that it is okay to write
      $$sum_ja_jdelta_ij=a_itag1$$
      or
      $$sum_ia_idelta_ij=a_jtag2$$



      Expression $(1)$ matches $(mathrmA)$ and expression $(2)$ matches $(mathrmB)$




      So does this mean that switching the order of the indices of the Kronecker delta (within a summation) has no effect on the result (the RHS), or am I missing something?










      share|cite|improve this question













      In my notes I have that




      $$sum_m a_m delta_nm=a_1delta_n1+a_2delta_n2+a_3delta_n3+cdots=a_ntagA$$




      Is this really correct?



      I thought that for the Kronecker delta the first index must match the summation index. For this reason, I thought that the expression $(mathrmA)$ should written as



      $$sum_colorbluem a_m delta_colorbluemn=a_1delta_1n+a_2delta_2n+a_3delta_3n+cdots=a_ntagB$$



      Just to make my argument clear, I have made the color of the indices match.



      After looking at this page on the Kronecker delta I know that it is okay to write
      $$sum_ja_jdelta_ij=a_itag1$$
      or
      $$sum_ia_idelta_ij=a_jtag2$$



      Expression $(1)$ matches $(mathrmA)$ and expression $(2)$ matches $(mathrmB)$




      So does this mean that switching the order of the indices of the Kronecker delta (within a summation) has no effect on the result (the RHS), or am I missing something?







      sequences-and-series algebra-precalculus summation index-notation kronecker-delta






      share|cite|improve this question













      share|cite|improve this question











      share|cite|improve this question




      share|cite|improve this question










      asked Sep 10 at 18:55









      BLAZE

      5,93692653




      5,93692653




















          2 Answers
          2






          active

          oldest

          votes

















          up vote
          1
          down vote



          accepted










          The Kronecker symbol $delta_ij$ is used to say $delta_ii=1$ and $delta_ij=0$ if $i neq j$.



          This is independent of any summation. Kronecker symbol can be used in general. That being said, the evaluation of formula $(A)$ is perfectly correct as the only term that is not vanishing is when index $m$ is equal to $n$.






          share|cite|improve this answer
















          • 1




            So is it okay to switch the order of the indices or not (the question I'm asking)?
            – BLAZE
            Sep 10 at 19:06






          • 1




            I updated the answer. In the specific case of sum $(A)$, you can indeed switch the indices.
            – mathcounterexamples.net
            Sep 10 at 19:10


















          up vote
          1
          down vote













          By its definition, $delta_i,j=delta_j,i,$ [one can drop the commas as you have] one can switch the indices anywhere, in a sum or otherwise.






          share|cite|improve this answer




















            Your Answer




            StackExchange.ifUsing("editor", function ()
            return StackExchange.using("mathjaxEditing", function ()
            StackExchange.MarkdownEditor.creationCallbacks.add(function (editor, postfix)
            StackExchange.mathjaxEditing.prepareWmdForMathJax(editor, postfix, [["$", "$"], ["\\(","\\)"]]);
            );
            );
            , "mathjax-editing");

            StackExchange.ready(function()
            var channelOptions =
            tags: "".split(" "),
            id: "69"
            ;
            initTagRenderer("".split(" "), "".split(" "), channelOptions);

            StackExchange.using("externalEditor", function()
            // Have to fire editor after snippets, if snippets enabled
            if (StackExchange.settings.snippets.snippetsEnabled)
            StackExchange.using("snippets", function()
            createEditor();
            );

            else
            createEditor();

            );

            function createEditor()
            StackExchange.prepareEditor(
            heartbeatType: 'answer',
            convertImagesToLinks: true,
            noModals: false,
            showLowRepImageUploadWarning: true,
            reputationToPostImages: 10,
            bindNavPrevention: true,
            postfix: "",
            noCode: true, onDemand: true,
            discardSelector: ".discard-answer"
            ,immediatelyShowMarkdownHelp:true
            );



            );













             

            draft saved


            draft discarded


















            StackExchange.ready(
            function ()
            StackExchange.openid.initPostLogin('.new-post-login', 'https%3a%2f%2fmath.stackexchange.com%2fquestions%2f2912231%2fdoes-changing-the-order-of-the-indices-of-the-kronecker-delta-within-a-summation%23new-answer', 'question_page');

            );

            Post as a guest






























            2 Answers
            2






            active

            oldest

            votes








            2 Answers
            2






            active

            oldest

            votes









            active

            oldest

            votes






            active

            oldest

            votes








            up vote
            1
            down vote



            accepted










            The Kronecker symbol $delta_ij$ is used to say $delta_ii=1$ and $delta_ij=0$ if $i neq j$.



            This is independent of any summation. Kronecker symbol can be used in general. That being said, the evaluation of formula $(A)$ is perfectly correct as the only term that is not vanishing is when index $m$ is equal to $n$.






            share|cite|improve this answer
















            • 1




              So is it okay to switch the order of the indices or not (the question I'm asking)?
              – BLAZE
              Sep 10 at 19:06






            • 1




              I updated the answer. In the specific case of sum $(A)$, you can indeed switch the indices.
              – mathcounterexamples.net
              Sep 10 at 19:10















            up vote
            1
            down vote



            accepted










            The Kronecker symbol $delta_ij$ is used to say $delta_ii=1$ and $delta_ij=0$ if $i neq j$.



            This is independent of any summation. Kronecker symbol can be used in general. That being said, the evaluation of formula $(A)$ is perfectly correct as the only term that is not vanishing is when index $m$ is equal to $n$.






            share|cite|improve this answer
















            • 1




              So is it okay to switch the order of the indices or not (the question I'm asking)?
              – BLAZE
              Sep 10 at 19:06






            • 1




              I updated the answer. In the specific case of sum $(A)$, you can indeed switch the indices.
              – mathcounterexamples.net
              Sep 10 at 19:10













            up vote
            1
            down vote



            accepted







            up vote
            1
            down vote



            accepted






            The Kronecker symbol $delta_ij$ is used to say $delta_ii=1$ and $delta_ij=0$ if $i neq j$.



            This is independent of any summation. Kronecker symbol can be used in general. That being said, the evaluation of formula $(A)$ is perfectly correct as the only term that is not vanishing is when index $m$ is equal to $n$.






            share|cite|improve this answer












            The Kronecker symbol $delta_ij$ is used to say $delta_ii=1$ and $delta_ij=0$ if $i neq j$.



            This is independent of any summation. Kronecker symbol can be used in general. That being said, the evaluation of formula $(A)$ is perfectly correct as the only term that is not vanishing is when index $m$ is equal to $n$.







            share|cite|improve this answer












            share|cite|improve this answer



            share|cite|improve this answer










            answered Sep 10 at 19:03









            mathcounterexamples.net

            1




            1







            • 1




              So is it okay to switch the order of the indices or not (the question I'm asking)?
              – BLAZE
              Sep 10 at 19:06






            • 1




              I updated the answer. In the specific case of sum $(A)$, you can indeed switch the indices.
              – mathcounterexamples.net
              Sep 10 at 19:10













            • 1




              So is it okay to switch the order of the indices or not (the question I'm asking)?
              – BLAZE
              Sep 10 at 19:06






            • 1




              I updated the answer. In the specific case of sum $(A)$, you can indeed switch the indices.
              – mathcounterexamples.net
              Sep 10 at 19:10








            1




            1




            So is it okay to switch the order of the indices or not (the question I'm asking)?
            – BLAZE
            Sep 10 at 19:06




            So is it okay to switch the order of the indices or not (the question I'm asking)?
            – BLAZE
            Sep 10 at 19:06




            1




            1




            I updated the answer. In the specific case of sum $(A)$, you can indeed switch the indices.
            – mathcounterexamples.net
            Sep 10 at 19:10





            I updated the answer. In the specific case of sum $(A)$, you can indeed switch the indices.
            – mathcounterexamples.net
            Sep 10 at 19:10











            up vote
            1
            down vote













            By its definition, $delta_i,j=delta_j,i,$ [one can drop the commas as you have] one can switch the indices anywhere, in a sum or otherwise.






            share|cite|improve this answer
























              up vote
              1
              down vote













              By its definition, $delta_i,j=delta_j,i,$ [one can drop the commas as you have] one can switch the indices anywhere, in a sum or otherwise.






              share|cite|improve this answer






















                up vote
                1
                down vote










                up vote
                1
                down vote









                By its definition, $delta_i,j=delta_j,i,$ [one can drop the commas as you have] one can switch the indices anywhere, in a sum or otherwise.






                share|cite|improve this answer












                By its definition, $delta_i,j=delta_j,i,$ [one can drop the commas as you have] one can switch the indices anywhere, in a sum or otherwise.







                share|cite|improve this answer












                share|cite|improve this answer



                share|cite|improve this answer










                answered Sep 10 at 19:08









                coffeemath

                1,4431313




                1,4431313



























                     

                    draft saved


                    draft discarded















































                     


                    draft saved


                    draft discarded














                    StackExchange.ready(
                    function ()
                    StackExchange.openid.initPostLogin('.new-post-login', 'https%3a%2f%2fmath.stackexchange.com%2fquestions%2f2912231%2fdoes-changing-the-order-of-the-indices-of-the-kronecker-delta-within-a-summation%23new-answer', 'question_page');

                    );

                    Post as a guest













































































                    這個網誌中的熱門文章

                    Why am i infinitely getting the same tweet with the Twitter Search API?

                    Is there any way to eliminate the singular point to solve this integral by hand or by approximations?

                    Strongly p-embedded subgroups and p-Sylow subgroups.